¿Cuál es el papel del tensor FμνFμνF_{\mu\nu} en QED?

En Electrodinámica Clásica la cantidad físicamente importante es el tensor electromagnético F Ω 2 ( METRO ) dónde METRO es espacio-tiempo.

Resulta que desde d F = 0 por el lema de Poincaré (asumiendo que METRO es contráctil), debe haber A Ω 1 ( METRO ) tal que F = d A .

Este A se utiliza entonces para simplificar las cosas porque es más sencillo de calcular A y A rendimientos directos F . Además, F es más fundamental, ya que F puede existir incluso si METRO no es contraible mientras que en este caso A no se garantiza que exista ya que no se aplica el lema de Poincaré. Más que eso como he dicho, lo físico es F , mientras A no es. Una forma de ver esto es que podemos agregar cualquier d ϕ a A , porque d ( A + d ϕ ) = d A y F no se altera y por lo tanto la física no se cambia.

Ahora vayamos a QED. En QED resulta que parece (al menos por el tratamiento que usan los libros que estoy leyendo) que el objeto importante es A . El campo asociado al fotón es A , el campo que se cuantifica es A y A produce las reglas de Feynman.

El campo F aparece en el lagrangiano, pero está escrito directamente en términos de A y todo se hace con A . Casi nunca el campo F parece ser utilizado en QED.

¿Porqué es eso? Si aun de la EM clásica, sabemos que la cosa física es F y A es solo algo para hacer la vida más fácil en los cálculos que no tienen un significado físico en sí mismo, por qué en QED parece que el objeto importante es A ? En ese caso, lo que acaba siendo el papel de F en QED?

No estoy seguro de lo que realmente estás pidiendo aquí. Si escribe electromagnetismo clásico en la formulación lagrangiana, entonces también tiene que escribir todo en términos de A , ya que el Lagrangiano no funciona si consideras F como la variable dinámica. Entonces, ¿cuál es la pregunta que es específica de la teoría cuántica aquí?

Respuestas (6)

Incluso en la electrodinámica clásica, al formular la teoría como un principio de acción, debe escribir la acción en términos de A . De lo contrario, no podrá recuperar las ecuaciones de Maxwell. con una acción F 2 , si F es el campo con respecto al cual debemos tomar la variación de la acción, solo podemos obtener ecuaciones de orden cero en el tiempo para F , lo cual es obviamente absurdo debido a la corrección de Maxwell a la ley de Ampere, × B = j + t mi -- primer orden en el tiempo.

Tampoco podemos conseguir B = 0 o t B = × mi sin agregarlas como restricciones al Lagrangiano, sino porque estas restricciones son precisamente d F = 0 , eso es lo mismo que declarar F = d A en subconjuntos contráctiles.

Ahora, la transformación A A + d ϕ se llama transformación de calibre. F y por lo tanto la acción F 2 es invariable bajo transformaciones de calibre, por lo que en realidad no necesitamos que exista un potencial de calibre global. Uno existe en cada subconjunto contráctil, si los dominios de A 1 y A 2 superposición, luego en la superposición A 1 y A 2 son transformaciones de calibre entre sí, y no importa cuál usemos.

Podemos "elevar" esto a una mejor formulación de la electrodinámica mediante el uso de haces de fibras. Desde este punto de vista, solo hay un potencial de calibre, pero vive en un "paquete principal" en el espacio-tiempo. El Lagrangiano es entonces una función de este paquete. Al elegir una "sección" del paquete, obtenemos el potencial en un indicador específico como un objeto en el espacio-tiempo, pero no se garantiza que tales secciones existan globalmente, por razones topológicas.

La forma precisa de hacer esto requiere un tratamiento del tamaño de un libro, más o menos. Puedo recomendar

  • Baez y Muniain, Gauge Fields, Knots, and Gravity .
  • Naber, Campos de topología, geometría y calibre , 2 vols.

con experiencia en geometría diferencial de, por ejemplo,

  • Lee, Colectores y Geometría Diferencial

También puede encontrar material útil en

  • Penrose, El camino a la realidad .
  • Nakahara, Geometría, topología y física (Este libro no me gustó mucho, pero mucha gente lo menciona...)
Secundo tu opinión sobre el trabajo de Nakahara; parece más una enciclopedia, tratando de abarrotar muchos temas sin la debida profundidad que ameritan.
Estoy entre las personas que encuentran en Nakahara un recurso fundamental para la física :p
Debo estar en una minoría, encontré a Nakahara un poco como un esfuerzo de copiar y pegar, aún así supongo que funciona para los crammers;).

Entonces, en cuanto a la primera pregunta, ¿por qué A tan fundamental en la Mecánica Cuántica? , esto se debe a que el momento es tan fundamental en QM, y el QED Lagrangian especifica que el momento canónico necesariamente contiene algún vector potencial.

Quizás el lugar más obvio donde esto causa un efecto cuántico real es en los llamados anillos AB (anillos de "Aharonov-Bohm"). Un anillo AB es simplemente un camino por el que viaja un electrón, que tiene la forma de un camino en forma de anillo, pero también con dos terminales agregados en el anillo: una entrada y una salida, para lo cual queremos saber la amplitud de transmisión. . Podemos describir cada uno de estos nodos de entrada y salida con una matriz de dispersión muy parecida a la de un divisor de haz; pero para los "brazos" que giran alrededor del círculo, el vector potencial se vuelve extremadamente importante porque agrega una fase mecánica cuántica a cada brazo. Podemos restar algo de esta fase, pero lo que no podemos eliminar en absoluto es la fase cuántica añadida al atravesar todo el círculo: esto es proporcional al flujo del campo a través del bucle.

Aquí está el resultado: al ajustar este potencial vectorial, incluso si el campo magnético se mantiene constante en cero en el bucle real , vemos efectos de interferencia debido a la modulación del potencial vectorial de las fases de estos dos brazos. Y es por eso que, en cierto sentido, este campo de potencial vectorial también se ve ahora como muy "fundamental": esta descripción es mucho más fácil si reconoce A que si tratas de resolverlo desde un F -perspectiva céntrica.

Otro punto que no se plantea en las otras dos buenas respuestas es que la transformación de calibre que dice hace F más físico que A es en realidad un ingrediente esencial en la elaboración de las teorías cuánticas de campos (QFT). El ejemplo más simple es la electrodinámica cuántica (QED). Comencemos con el Lagrangiano para un campo de masa libre de fermiones metro (el γ m son cuatro matrices que actúan sobre el espinor ψ y su conjugado ψ ¯ y la suma de Einstein se utilizará en todo momento).

L F = ψ ¯ ( i γ m m metro ) ψ

Una transformación ψ ( X ) mi i q Λ ψ ( X ) , con una constante Λ , deja la invariante lagrangiana, como ψ ¯ ( X ) mi i q Λ ψ ( X ) . Inmediatamente reconoces que estas transformaciones son una representación grupal de tu ( 1 ) . Pero la receta que ha demostrado ser útil para escribir QFT Lagrangian es requerir una simetría de calibre local , es decir, invariancia bajo ψ ( X ) mi i q Λ ( X ) ψ ( X ) .

L F Sin embargo, no es invariante bajo esta simetría de calibre local:

L F L F i q ψ ¯ ( γ m m Λ ) ψ

Pero luego, cuando agregamos la interacción entre el campo fermiónico y el campo electromagnético,

L I = q ψ ¯ γ m ψ A m ,

debería ser obvio que el término i q ψ ¯ ( γ m m Λ ) ψ puede cancelarse mediante una transformación de calibre de A m :

A m A m + m Λ

Moral: la transformación de calibre de A juega un papel crucial en QED. Transformaciones de calibre más complejas de ψ , representaciones de grupos más complejas que tu ( 1 ) son la base del Modelo Estándar. Por ejemplo, S tu ( 3 ) para QCD. Y alguna representación arcana de S tu ( 2 ) × tu ( 1 ) por la teoría electrodébil. De paso, en ese último caso, un término de masa explícito para el fermión rompería la invariancia de calibre local, por lo que tendría que omitir eso y traer el mecanismo de Higgs, pero esta es otra historia que traigo solo para no t empujar falsedades!

"... por qué en QED parece que el objeto importante es A ?..."

Esto es cierto, y la razón de esto es que las interacciones EM estáticas tienen 1 / r 2 comportamiento como r va al infinito. Realmente, la declaración significa que el propagador de la interacción EM debe tener 1 / pag 2 forma, que (teniendo en cuenta dos polarizaciones y la repulsión de cargas del mismo signo) solo puede describirse mediante un campo de tipo vectorial sin masa. El F m v tensor daría un comportamiento diferente del potencial estático.

"... En ese caso, lo que termina siendo el papel de F en QED?..."

1) A m ya que un vector de 4 Lorentz es completamente afísico (exceptuando algunas situaciones), y no solo por la variación de calibre. Una razón más importante es que no puede representar partículas definidas matemáticamente como representaciones irreducibles del grupo de Poincaré con masa cero. Esto es crucial siempre que se considere que la simetría de Poincaré es la simetría de nuestro mundo. El verdadero campo covariante de Poincaré que representa los fotones es F m v .

2) Como el ED clásico, QED se basa en la simetría de calibre (que es más bien la redundancia de calibre en la teoría cuántica, pero esto no es importante en este momento). Por lo tanto, todas las cantidades observadas que dependen del campo EM siempre se expresan en términos de F m v . En particular, si partimos de la complicada teoría de calibre que incluye F m v , campos de materia y A m mismo, y luego integrar los campos de materia, la acción efectiva resultante contendrá solo F m v .

el papel de es escribir la Ecuación de Maxwell en cuatro dimensiones, ahora los cuatro vectores de energía-momentum PAG m , así consideramos el cuadrivector de potencial A m = ( ϕ , A ) , La cantidad A se denomina potencial vectorial o campo de calibre, donde la intensidad del campo se define mediante:\ F m v = m A v v A m

Las ecuaciones de Maxwell se pueden escribir en términos del tensor antisimétrico F m v definido por:\ F m v = m A v v A m = ( 0 mi X mi y mi z mi X 0 B z B y mi y B z 0 B X mi z B y B X 0 ) \ Es evidente que el campo electromagnético es un campo tensorial. \ Por lo tanto encontramos:

F 0 i = O A i i A 0 = A i t + i ϕ = mi i
F i j = i A j j A i = ε i j k ( × A ) = ε i j k B k
\ Ex:\
F 01 = A 1 X 0 A 0 X 1 = mi X
Como este ejemplo, puedes reescribir todas las ecuaciones de Maxwelle. También podemos escribir la ecuación de movimiento por: Las ecuaciones de movimiento que se derivan del Lagrangiano
L = 1 4 F m v F m v
que mencionamos son:\
m ( L ( m A v ) ) = m F m v = 0.
A resulta de una corrección a las ecuaciones de Maxwell en la forma:
B = × A
y
mi = ϕ A t
L q mi D = ψ ¯ [ γ m ( i m q A m ) metro ] ψ 1 4 F m v F m v j m A m

Está confundiendo ser "físico", en el sentido de ser invariante de calibre, con ser canónico (es decir, ser el q en la ecuación de Euler-Lagrange). Mientras F es invariante, A es canónico. Necesitamos un F 2 término en la acción porque sin derivadas de A su impulso se desvanece y no podemos formular un formalismo hamiltoniano interesante.

No es sorprendente que podamos cambiar el campo canónico con una transformación que conserva la acción, si sabes algo sobre las transformaciones canónicas en la mecánica clásica. Tampoco es sorprendente que necesitemos una dinámica A -campo, ya que sin un control en el caparazón de cómo varía, la transformación de calibre no funcionará. Lo que quiero decir con eso es que no podemos decir muy bien A es una constante puesta a mano, ya que las transformaciones de calibre arbitrarias no preservarán eso.